Vous êtes sur la page 1sur 96

LECTURE 1-5

ANALYTICAL REASONING

Questions 1- 3

Three men (Tom, Peter and Jack) and three women (Eliza, Anne and Karen) are
spending a few months at a hillside. They are to stay in a row of nine cottages,
each one living in his or her own cottage. There are no others staying in the
same row of houses.

1. Anne, Tom and Jack do not want to stay in any cottage, which is at the
end of the row.
2. Eliza and Anne are unwilling to stay besides any occupied cottage..
3. Karen is next to Peter and Jack.
4. Between Anne and Jack's cottage there is just one vacant house.
5. None of the girls occupy adjacent cottages.
6. The house occupied by Tom is next to an end cottage.

1. Which of the above statements can be said to have been derived from two
other statements ?
A. Statement 1
B. Statement 2
C. Statement 3
D. Statement 5
E. Statement 6

Ans : D

2. How many of them occupy cottages next to a vacant cottage ?


A. 2
B. 3
C. 4
D. 5
E. 6

Ans : C

3. Which among these statement(s) are true ?

I. Anne is between Eliza and Jack.


II. At the most four persons can have occupied cottages on either side
of them. .
III. Tom stays besides Peter.

A. I only

1
B. II only
C. I and III only
D. II and III only
E. I, II and III

Ans : C

Questions 4 - 7

An employee has been assigned the task of allotting offices to six of the staff
members. The offices are numbered 1 - 6. The offices are arranged in a row and
they are separated from each other by six foot high dividers. Hence voices,
sounds and cigarette smoke flow easily from one office to another.

Miss Robert's needs to use the telephone quite often throughout the day. Mr.
Mike and Mr. Brown need adjacent offices as they need to consult each other
often while working. Miss. Hardy, is a senior employee and has to be allotted the
office number 5, having the biggest window. .

Mr. Donald requires silence in the offices next to his. Mr. Tim, Mr. Mike and Mr.
Donald are all smokers. Miss Hardy finds tobacco smoke allergic and
consecutively the offices next to hers to be occupied by non-smokers.

Unless specifically stated all the employees maintain an atmosphere of silence


during office hours.

4. The ideal candidate to occupy the office furthest from Mr. Brown would be
A. Miss Hardy
B. Mr. Mike
C. Mr. Tim
D. Mr. Donald
E. Mr. Robert

Ans : D

5. The three employees who are smokers should be seated in the offices.
A. 1, 2 and 4
B. 2, 3 and 6
C. 1, 2 and 3
D. 1, 2 and 3
E. 1, 2 and 6

Ans : D

6. The ideal office for Mr. Mike would be.


A. 2

2
B. 6
C. 1
D. 3
E. 4

Ans : D

7. In the event of what occurrence, within a period of one month since the
assignment of the offices, would a request for a change in office be put
forth by one or more employees ?
A. Mr. Donald quitting smoking.
B. The installation of a noisy teletype machine by Miss Hardy in her
office.
C. Mr. Robert's needing silence in the office (s) next to her own. .
D. Mr. Brown suffering from laryngitis.
E. Mr. Tim taking over the duties formerly taken care of by Miss.
Robert. .

Ans : E

In an experiment conducted at a laboratory, 160 white mice were injected with


Serum D. 160 other white mice were injected with a harmless sugar solution .In
two weeks time 39% of the white mice, who were injected with Serum D
contracted the highly contagious and often fatal disease, jungle fever. Hence, it
can be concluded that jungle fever is caused by some elements similar to the
elements in Serum D.

8. The above discussion would be weakened most severely in case it is


shown that
A. People contracting jungle fever are usually the victims of the bite of
the South American Lesser Hooded Viper.
B. One among the 160 white mice had already contracted jungle fever
prior to the laboratory experiment.
C. The natural habitats of white mice does not contain any of the
elements found in Serum D.
D. The scientists administered the injections being ignorant of the
contents of the solutions used.
E. The 160 white mice used in the laboratory experiment were kept
isolated from each other.

Ans : B

9. The above argument would be highly empowered in case it were shown


that:
A. Some of the elements in Serum D are extracted from the root of a
certain poisonous jungle wildflower.

3
B. Within a period of two weeks about 40% of the white mice, who
were injected with a harmless sugar solution also contracted jungle
fever.
C. Almost all the white mice died within a period of two days after the
first symptoms appeared.
D. Normally the rate of jungle fever among white mice is less than
0.01%.
E. Invariably the blood of the victims of jungle fever victims contains a
high level of a certain toxic substance also found in serum D.

Ans : E

10. Distribution of leaflets and delivering speeches on government property


should be outlawed. Radicals and fanatics have no right to use public
property when peddling their unsavory views.

The argument above is based on the postulate

A. The general public has a special concern in the free exchange of


different political views.
B. Radicals and fanatics prefer the use of public property while
propagating their viewpoint.
C. Every person who hands out leaflets and delivers speeches is a
radical or fanatic.
D. Legal constraints which are applicable to one group need not be
equally applicable to all.
E. Any political activity, which hinders the proper functioning of the
government should not be protected by the law.

Ans : C

Questions 11 - 12

Successfully adjusting to one's environment leads to happiness. War at a


universal level war destroys the weaker people, who are the most unable to
adjust to their environment. Thus, war at the universal level puts weaklings out of
their misery and allows more space for their predators to enjoy life in a better
manner. As those actions have to be performed, which maximize the level of
happiness of the greatest number, war at a universal level should take place.

11. What response would the author of the above discussion come up with, in
the case of the objection that the weaklings far exceed strong people?
I. He would respond with the statement that the person making the
objection is a weakling.
II. He would respond by saying that weaklings will be miserable no
matter what happens.

4
III. He would respond with the statement that the strong would be
frustrated if the weaklings are destroyed.

A. I only
B. II only
C. III only
D. I and II only
E. II and III only

Ans : E

12. The author's discussion would be greatly if he agreed to which of the


following?

I. Technology could change the environment.


II. War at the universal level would be an integral part of the
environment.
III. It is possible for the strong to survive without suppressing the weak.

A. I only
B. II only
C. III only
D. I and III only
E. I, II and III only

Ans : A

Questions 13 - 15

Come back with us to the real America leaving behind the turmoil of civilization.
The real America is still inhabited by the eagle, the buffalo, the mountain lion and
elk; it is still spacious, sprawling and majestic. Experience the freedom and
serenity still to be found in

13. Choose the best option to complete the above statement:


A. the natural beauty of our land
B. the fascinating urban centers
C. the wild terrain of Africa
D. one's own subconscious
E. the great sprawling cities of the Southwest

Ans : A

14. The above paragraph is most likely to appear in which of the following?
A. A Hunter's Guide to The United States
B. Exploring the Great Outdoors

5
C. The Quiet Beauty of Alaska
D. How the Eagle Became Extinct
E. Returning to America

Ans : D

15. When I am elected, I will work towards effecting those changes for which I
have been fighting all these years. We will work together to do away with
the bureaucratic bogs which have existed ever since my opponent took
office. Everyone of you knows what I stand for; I invite my opponent to ...

For completion of the above statement choose the best option:

A. hand in his resignation graciously


B. make his stance clear
C. stop lying to the public
D. get our city more federal aid
E. extend his support to me

Ans : B

16. We can never make our beliefs regarding the world certain. Even scientific
theory of a most rigorous and well-confirmed nature is likely to change
over a decade or even tomorrow. If we refuse to even try to understand,
then it is like resigning from the human race. Undoubtedly life of an
unexamined kind is worth living in other respects--as it is no mean thing to
be a vegetable or an animal. It is also true that a man wishes to see this
speculative domain beyond his next dinner.

From the above passage it is clear that the author believes that

A. men would not do well to speculate


B. progress in the scientific field is impossible
C. one should live life with the dictum 'what will be will be'
D. men should ignore their animal needs
E. men are different from animals as far as their reasoning abilities are
concerned.

Ans : E

Questions 17-21

Two or more essences out of a stock of five essences-- L, M, N, O, and P are


used in making all perfumes by a manufacturer. He has learned that for a blend
of essences to be agreeable it should comply with all the rules listed below.

6
A perfume containing L, should also contain the essence N, and the quantity of N
should be twice as that of L.
A perfume containing M, must also have O as one of its components and they
should be in equal proportion.
A single perfume should never contain N as well as O.
O and P should not be used together.
A perfume containing the essence P should contain P in such a proportion that
the total amount of P present should be greater than the total amount of the other
essence or essences used.

17. Among the following which is an agreeable formula for a perfume?


A. One part L, one part P
B. Two parts M, two parts L
C. Three parts N, three parts L
D. Four parts O, four parts M
E. Five parts P, five parts M

Ans : D

18. Adding more amount of essence N will make which of the following
perfumes agreeable?
A. One part L, one part N, five parts P
B. Two parts M, two parts N, two parts P
C. One part M, one part N, one part P
D. Two parts M, one part N, four parts P
E. Two parts N, one part O, three parts P

Ans : A

19. Among the following, the addition of which combination would make an
unagreeable perfume containing two parts N and one part P agreeable?

(A) One part L (B) One part M (C) Two parts N (D) One part O (E)
Two parts P

Ans : E

20. Among the following which combination cannot be used together in an


agreeable perfume containing two or more essences?
A. L and M
B. L and N
C. L and P
D. M and O
E. P and N

Ans : A

7
21. Among the below mentioned formulas, which can be made agreeable by
the eliminating some or all of one essence ?
A. One part L, one part M, one part N, four parts P
B. One part L, two parts N, one part O, four parts P
C. One part L, one part M, one part O, one part P
D. Two parts L, two parts N, one part O, two parts P
E. Two parts M, one part N, two parts O, three parts P

Ans : B

Questions 13 - 15

Come back with us to the real America leaving behind the turmoil of civilization.
The real America is still inhabited by the eagle, the buffalo, the mountain lion and
elk; it is still spacious, sprawling and majestic. Experience the freedom and
serenity still to be found in

13. Choose the best option to complete the above statement:


A. the natural beauty of our land
B. the fascinating urban centers
C. the wild terrain of Africa
D. one's own subconscious
E. the great sprawling cities of the Southwest

Ans : A

14. The above paragraph is most likely to appear in which of the following?
A. A Hunter's Guide to The United States
B. Exploring the Great Outdoors
C. The Quiet Beauty of Alaska
D. How the Eagle Became Extinct
E. Returning to America

Ans : D

15. When I am elected, I will work towards effecting those changes for which I
have been fighting all these years. We will work together to do away with
the bureaucratic bogs which have existed ever since my opponent took
office. Everyone of you knows what I stand for; I invite my opponent to ...

For completion of the above statement choose the best option:

A. hand in his resignation graciously


B. make his stance clear
C. stop lying to the public
D. get our city more federal aid

8
E. extend his support to me

Ans : B

16. We can never make our beliefs regarding the world certain. Even scientific
theory of a most rigorous and well-confirmed nature is likely to change
over a decade or even tomorrow. If we refuse to even try to understand,
then it is like resigning from the human race. Undoubtedly life of an
unexamined kind is worth living in other respects--as it is no mean thing to
be a vegetable or an animal. It is also true that a man wishes to see this
speculative domain beyond his next dinner.

From the above passage it is clear that the author believes that

A. men would not do well to speculate


B. progress in the scientific field is impossible
C. one should live life with the dictum 'what will be will be'
D. men should ignore their animal needs
E. men are different from animals as far as their reasoning abilities are
concerned.

Ans : E

Questions 17-21

Two or more essences out of a stock of five essences-- L, M, N, O, and P are


used in making all perfumes by a manufacturer. He has learned that for a blend
of essences to be agreeable it should comply with all the rules listed below.

A perfume containing L, should also contain the essence N, and the quantity of N
should be twice as that of L.
A perfume containing M, must also have O as one of its components and they
should be in equal proportion.
A single perfume should never contain N as well as O.
O and P should not be used together.
A perfume containing the essence P should contain P in such a proportion that
the total amount of P present should be greater than the total amount of the other
essence or essences used.

17. Among the following which is an agreeable formula for a perfume?


A. One part L, one part P
B. Two parts M, two parts L
C. Three parts N, three parts L
D. Four parts O, four parts M
E. Five parts P, five parts M

9
Ans : D

18. Adding more amount of essence N will make which of the following
perfumes agreeable?
A. One part L, one part N, five parts P
B. Two parts M, two parts N, two parts P
C. One part M, one part N, one part P
D. Two parts M, one part N, four parts P
E. Two parts N, one part O, three parts P

Ans : A

19. Among the following, the addition of which combination would make an
unagreeable perfume containing two parts N and one part P agreeable?

(A) One part L (B) One part M (C) Two parts N (D) One part O (E)
Two parts P

Ans : E

20. Among the following which combination cannot be used together in an


agreeable perfume containing two or more essences?
A. L and M
B. L and N
C. L and P
D. M and O
E. P and N

Ans : A

21. Among the below mentioned formulas, which can be made agreeable by
the eliminating some or all of one essence ?
A. One part L, one part M, one part N, four parts P
B. One part L, two parts N, one part O, four parts P
C. One part L, one part M, one part O, one part P
D. Two parts L, two parts N, one part O, two parts P
E. Two parts M, one part N, two parts O, three parts P

Ans : B

Questions 26 - 31

Nine individuals - Z, Y, X, W, V, U, T, S and R - are the only candidates, who can


serve on three committees-- A, B and C, and each candidate should serve on
exactly one of the committees.

10
Committee A should consist of exactly one member more than committee B.
It is possible that there are no members of committee C.
Among Z, Y and X none can serve on committee A.
Among W, V and U none can serve on committee G.
Among T, S and R none can serve on committee C.

26. In case T and Z are the individuals serving on committee B, how many of
the nine individuals should serve on committee C?
A. 3
B. 4
C. 5
D. 6
E. 7

Ans : B

27. Of the nine individuals, the largest number that can serve together on
committee C is
A. 9
B. 8
C. 7
D. 6
E. 5

Ans : D

28. In case R is the only individual serving on committee B, which among the
following should serve on committee A?
A. W and S
B. V and U
C. V and T
D. U and S
E. T and S

Ans : E

29. In case any of the nine individuals serves on committee C, which among
the following should be the candidate to serve on committee A?
A. Z
B. Y
C. W
D. T
E. S

Ans : C

11
30. In case T, S and X are the only individuals serving on committee B, the
total membership of committee C should be:
A. Z and Y
B. Z and W
C. Y and V
D. Y and U
E. X and V

Ans : A

31. Among the following combinations which could constitute the membership
of committee C?
A. Y and T
B. X and U
C. Y, X and W
D. W, V and U
E. Z, X, U and R

Ans : B

Questions 32 - 34

(M, N, O and P are all different individuals)

I. M is the daughter of N.
II. N is the son of O
III. O is the father of P.

32. Among the following statements, which is true ?


A. O is the uncle of M.
B. P and N are brothers
C. M is the daughter of P.
D. If B is the daughter of N, then M and B are sisters.
E. If C is the granddaughter of O, then C and M are sisters.

Ans : D

33. Which among the following statements is contradictory to the above


premises?
A. P is the father of M.
B. O has three children.
C. M has one brother.
D. M is the granddaughter of O.
E. Another party C, could be the mother of M.

Ans : A

12
34. If B is the son of N and B has one brother, D, then

I. M is the sister of D.
II. D and N are brothers.
III. O is the grandfather of D.

A. I only
B. II only
C. III only
D. I and III only
E. I and II only

Ans : D

Questions 35 - 41

The only people to attend a conference were four ship captains and the first
mates of three of those captains. The captains were L, M, N and O; the first
mates were A, D and G. Each person in turn delivered a report to the assembly
as follows:

Each of the first mates delivered their report exactly after his or her captain. The
first captain to speak was M, and captain N spoke after him.

35. Among the following which is not an appropriate order of delivered


reports?
A. M, A, N, G, O, L, D
B. M, D, N, G, L, O, A
C. M, N, A, L, D, O, G
D. M, N, A, O, D, L, G
E. M, N, G, D, O, L, A

Ans : E

36. In case L speaks after A, and A is the third of the first mates to speak,
then among the following statements which would be untrue?
A. O spoke immediately after G.
B. The order of the first four speakers was M, G, N, D.
C. O's first mate was present.
D. A was the fourth speaker after M.
E. The captains spoke in the order M, N, O, L.

Ans : D

37. Among the following statements which statement must be true?

13
A. In case the second speaker was a captain, the seventh speaker
was a first mate.
B. In case the second speaker was a first mate, the seventh speaker
was a captain.
C. In case the third speaker was a first mate, the seventh speaker was
a captain.
D. In case the third speaker was a captain, the seventh speaker was a
first mate.
E. In case the seventh speaker was a first mate, the first and third
speakers were captains.

Ans : A

38. In case A spoke immediately after L and immediately before O, and O was
not the last speaker, L spoke
A. second
B. third
C. fourth
D. fifth
E. sixth

Ans : C

39. In case G is M's first mate, D could be the person who spoke immediately
A. prior to T
B. prior to L
C. prior to V
D. after T
E. after V

Ans : D

40. In case A is the third of the first mates to speak, and L is the captain
whose first mate is not present, which among the following statements
must be true?
A. A spoke sometime before L.
B. D spoke sometime before O.
C. L spoke sometime before O.
D. O spoke sometime before L.
E. O spoke sometime before N.

Ans : B

41. Among the following statements, which would make M, D, N, G, L, O, A


the only possible sequence of speakers?
A. D is M's first mate; G is N's first mate; A is O's first mate.

14
B. D is M's first mate; G is N's first mate; A was the second to speak
after L.
C. The order of the first four speakers was M, D, N, G.
D. The order of the last three speakers was L, O, A.
E. The order in which the captains spoke was M, N, L, O.

Ans : B

Questions 42 - 45

1. The microbe A causes the contagious disease A.


2. The first symptoms appear after a period of two days since the microbe A
enters the body.
3. The microbe A is found in some flies and bees.
4. A fly bit Jack on Monday, February 6.
5. Nick worked with Jack the next day, Tuesday, February 7.

There were no other possibilities of exposure to Disease A.

42. In case Jack showed symptoms of Disease A, which of the following


statements would be true?
I. Jack contracted the Disease A from Nick.
II. Jack first noticed symptoms of Disease A on February 8.
III. The fly that Jack was bitten by was not a carrier of the microbe A.

A. I only
B. II only
C. III only
D. I and II only
E. I and III only

Ans : B

43. In case Nick displayed symptoms of the disease A, which among the
following would be true?
A. I only
B. II only
C. III only
D. II and III only
E. I, II and III

Ans : D

44. In case Jack displayed symptoms of Disease A, which would be true?

I. Jack was also bitten by a fly on February 5.

15
II. Jack was bitten by a mosquito which carried the microbe A.
III. Nick contracted Disease A from Jack.

1. I only
2. II only
3. III only
4. I and II only
5. II and III only

Ans : D

43. In case Nick displayed the symptoms of Disease A which would be true?

IV. Nick was bitten by a bee on February 6.


V. Nick ate food which contained the microbe A.
VI. Nick also worked with Jack on February 6.

1. I only
2. II only
3. III only
4. I and II only
5. I, II and III

Ans : A

Questions 46 - 47

The principal evil in today's society is selfishness. Everywhere we see people,


who are concerned only with themselves. Personal advancement is the only
motivating force in the world today. This does not mean that individuals are not
willing to help one another; on the contrary, _________. But, these are only
short-term occurrences which ultimately serve our long-term goal of personal
gain.

46. To fill in the blank in the above passage, select one of the options from the
below mentioned options:
1. we are always trying to undermine others’ endeavors.
2. my uncle Jeremy used to help me with my homework.
3. no one can be trusted, not even close friends.
4. our yearning for power prevents us from understanding our
existential purpose.
5. there are many occasions when we graciously offer our assistance.

Ans : E

16
47. Which among the following options would most strongly contradict the
author's attitude towards society?
1. The greatest strength of society is altruism.
2. The forces of good will ultimately triumph over evil.
3. Our short-term actions may ostensibly contradict our long-term
goals.
4. We must all learn the art of selfishness.
5. Morality is the bedrock of a growing community.

Ans : A

48. It can be proved by an example that our words are devoid of meaning as
they cannot be distinguished from their opposites. People think that they
are aware of the difference between the meanings of 'bald' and 'having
hair' Let us suppose that an average twenty-one year old has X strands of
hair on his or her head. We say that such a person is not bald but has
hair. But surely one hair less would make no difference, and a person with
X - 1 hairs on his or her head would be said to have hair. Suppose we go
on like this, decreasing one hair at a time, the result would be the same.
Then what difference would there be between someone who has one hair
and someone who has none? We call them both bald. Thus, we cannot
make a distinction between the terms 'bald' and 'having hair.'

Among the following statements, which statement best counters the


argument above?

A. The word 'bald' can be translated into other languages.


B. A word can have more than one meaning.
C. A word such as 'monkey' can be applied to several animals that
differ in some respects.
D. Words can lack precision without being meaningless.
E. People cannot think clearly without using words.

Ans : D

49. Virus M helps in controlling the population of gnats; they manage to do


this by killing the moth's larvae. Though the virus is always present in the
larvae, it is only every six or seven years that the virus seriously
decimates the numbers of larvae, greatly reducing the population of the
gnats. Scientists are of the opinion that the gnats, usually latent, are
activated only when the larvae experience biological stress.

In case the above mentioned scientists are correct, it can be inferred that
the decimation of gnat populations by the virus M could be most probably
activated by the following conditions?

17
A. A shift by the gnats from drought areas to a normal area occupied
by them
B. The resultant stress from defoliation of trees attacked by the gnats
for the second consecutive year.
C. Attacks on the larvae by all kinds by parasitic wasps and flies.
D. Starvation of the gnat larvae due to over population.
E. Spraying of gnat infested areas with laboratory - raised Virus M.

Ans : D

50. In a particular code, the digits from 0 to 9 inclusive are each represented
by a different letter of the alphabet, the letter always representing the
same digit. In case the following sum

BOPB
+SKB
--------
CVBQ

holds true when it is expressed in digits, which of the following cannot be


properly inferred:

A. B cannot be 0.
B. B must be less than 5.
C. Q must be even.
D. O + S must be greater than 8.
E. C must be greater than B by 1.

Ans : B

Questions 51 - 56

There are three on-off switches on a control panel A, B, and C. They have to be
changed from an initial setting to a second setting according to the following
conditions : In case only switch A is the switch on in the initial setting , then turn
on switch B.

In case switches A and B are the only switches on in the initial setting, then turn
on switch C. In case all the three switches are on initially setting, then turn off the
switch C. For any other initial setting, turn on all switches that are off and turn off
all switches, if any, that are on.

51. In case in the initial setting is the switches A and B are on and the switch
C is off, then what could be the second setting?
A. A on, B on, C on.
B. A on, B off, C on.

18
C. A on, B off, C off.
D. A off, B on, C off.
E. A off, B off, C on.

Ans : A

52. In case switch B is the only switch on in the initial setting, what must be
the second setting?
A. A on, B on, C on.
B. A on, B on, C off.
C. A on, B off, C on.
D. A off, B off, C on.
E. A off, B off, C off.

Ans : C

53. In case all the three switches are on in the second setting, which among
the following could have been the initial setting ?
A. A on, B on, C on.
B. A on, B on, C off.
C. A on, B off, C on.
D. A on, B off, C off.
E. A off, B on, C off

Ans : B

54. In case switch A is off in the second setting, which among the following
could have been the initial setting ?
A. A on, B on, C on.
B. A on, B on, C off.
C. A on, B off, C on.
D. A on, B off, C off.
E. A off, B on, C off.

Ans : C

55. In case only switch B is on in the second setting, which among the
following could have been the initial setting ?
A. A on, B on, C on.
B. A on, B off, C on.
C. A off, B on, C off.
D. A off, B off, C on.
E. A off, B off, C off.

Ans : B

19
56. Which among the following initial settings leads to a second setting, where
only one switch is off ?
A. A on, B on, C off.
B. A on, B off, C on.
C. A off, B on, C on.
D. A off, B on, C off.
E. A off, B off, C off.

Ans : D

Questions 57 - 59

A bus has exactly six stops on its route. The bus first stops at stop one and then
at stops two, three, four, five, and six respectively. After the bus leaves stop six,
the bus turns and returns to stop one and repeats the cycle. The stops are at six
building that are, in alphabetical order L, M, N, O, P, and Q.

P is the third stop.


M is the sixth stop.
The stop O is the stop immediately before Q.
N is the stop immediately before L.

57. In case N is the fourth stop, which among the following must be the stop
immediately before P ?
A. O
B. Q
C. N
D. L
E. M

Ans : B

58. In case L is the second stop, which among the following must be the stop
immediately before M ?
A. N
B. L
C. P
D. O
E. Q

Ans : E

59. In case a passenger gets on the bus at O, rides past one of the stops, and
gets off at P, which of the following must be true ?
A. O is stop one.
B. Q is stop three.

20
C. P is stop four.
D. N is stop five.
E. L is stop six.

Ans : A

60. A survey recently conducted revealed that marriage is fattening. The


survey found that on an average, women gained 23 pounds and men
gained 18 pounds during 13 years of marriage. The answer to which
among the following questions would be the most appropriate in
evaluating the reasoning presented in the survey ?
A. Why is the time period of the survey 13 years, rather than 12 or
14 ?
B. Did any of the men surveyed gain less than 18 pounds during the
period they were married ?
C. How much weight is gained or lost in 13 years by a single people of
comparable age to those studied in the survey ?
D. When the survey was conducted were the women as active as the
men?
E. Will the gains seen in the study be retained over the lifetimes of the
surveyed persons?

Ans : C

61. Classical works occupy a unique and peculiar position. They are
imperishable as cultural landmarks, but the views expressed in some of
the works are often superseded by newer views. But the classics survive
just like ancient castles, as imposing features of the landscape, yet
unsuited for habitation unless remodeled.

The principal point put forth in the above passage is that classical works.

A. Demand our respect and admiration even if their ideas are no


longer current.
B. Prove to be obstacles in the path of new ideas as they are unduly
respected.
C. Should be preserved and treasured as scholarly opinion keeps
changing and they will come into fashion again.
D. Lose their distinctive features as time passes.
E. Are not given due consideration by the younger generation.

Ans : A

62. Experts in the field of forestry are now questioning the policy of attempting
to extinguish all forest fires instead of allowing them to run their course

21
and die out on their own. The study of forest fires indicates that in the long
run, forest fires may in fact, be beneficial to the ecology as a whole.

Among the following, which statement would logically follow from one of
the ’observations’ referred to in the passage above ?

A. Most of the forest fires are causes due to natural reasons like
lightning rather than as a result of human carelessness.
B. Every year the number of people dying as they attempt to fight
forest fires is greater than the number of people who would have
died or suffered injury if the fires were allowed to run their own
course.
C. Accumulation of highly flammable dead brushwood and timber,
which could lead to massive uncontrollable fires is prevented by
frequent small forest fires.
D. The animal and plant life destroyed by forest fires seldom
represents endangered species.
E. Studies indicate that forest fires regularly occurred in the times prior
to human occupation of forest areas.

Ans : C

63. There is a theory that the dinosaurs inhabiting the earth were destroyed
when a huge heavenly body hit the earth. The impact of the heavenly
body when it struck the earth threw a mass of pulverized debris into the
atmosphere. This dust created a fog, which blocked the sunlight depleting
plant life. This resulted in the perishing of the plant-eating dinosaurs; the
meat-eating dinosaurs, who preyed on the plant-eaters inturn starved to
death.

Which among the following statements, would best support the above
mentioned theory, if true?

A. Enough dust has been sent into the atmosphere by modern


volcanic explosions to change the color of sunsets around the world
for several years.
B. Craters formed by heavenly bodies are plentiful in several areas of
the South west where many dinosaur fossils have also been found.
C. Studies conducted recently on the bone structure of dinosaurs
show that they were very agile and not ponderous awkward
animals as popularly believed.
D. The sedimentary rock strata of the earth formed from atmospheric
dust at the time the dinosaurs disappeared contains minerals and
other trace elements in proportions characteristic of the heavenly
bodies.

22
E. Many meat-and plant-eating species of mammals who coexisted
with the dinosaurs continue to exist today, although with a slightly
modified appearance.

Ans : D

Questions 64 - 70

Five educational films A, B, C, D, & E are to be shown to a group of students.


The films are to be shown in a particular order, which conforms to the following
conditions:

A must be shown earlier than C.


B must be shown earlier than D.
E should be the fifth film shown.

64. Which among the following is an acceptable order for showing the
educational films ?
A. A, C, B, D, E
B. A, C, D, E, B
C. B, D, C, A, E
D. B, D, E, A, C
E. E, B, C, A, D

Ans : D

65. In case C is shown earlier than E, which among the following will hold true
?
A. A is the first film shown.
B. B is the second film shown.
C. C is the third film shown.
D. D is the fifth film shown.
E. E is the fourth film shown.

Ans : D

66. In case D is to be shown earlier than A, then for which among the
following is there exactly one position from first through fifth in which it can
be scheduled to be shown ?
A. A
B. B
C. C
D. D
E. E

Ans : C

23
67. Which among the following is a pair of films that CANNOT both be shown
earlier than E ?
A. A and B
B. A and D
C. B and C
D. B and D
E. C and D

Ans : E

68. In case D and E are shown as far apart from each other as possible,
which among the following would be true ?
A. A is shown earlier than B.
B. B is shown earlier than C.
C. C is shown earlier than E.
D. D is shown earlier than A.
E. E is shown earlier than B.

Ans : E

69. In case B, D and E are to be shown one after the other in the given order,
the two positions from first to fifth in which A could possibly be shown are
A. first and second.
B. first and fourth.
C. second and third.
D. third and fifth.
E. fourth and fifth.

Ans : B

70. In case exactly one film is shown between A and C, and exactly one film is
shown between B and D, which among the following will hold true ?
A. B is the film shown between A and C.
B. C is the film shown between B and D.
C. E is the film shown between A and C.
D. D is the last film shown.
E. E is the first film shown.

Ans : E

71. Authorities concerned with mass transport in metropolitan cities are


struggling with deficits. Commuters complain about delays and
breakdowns, cuts in service, and fares higher than they are accustomed to
paying. For all these reasons and because the price of fuel is still not
prohibitive, the number of commuters using public transportation has
fallen, adding to the deficits.

24
Which among the following statements about the relationship between the
number of commuters using public transportation and the price of fuel is
best supported by the above passage?

A. With the rise in the price of fuel, there is a rise in the number of
commuters using public transportation.
B. Even if the price of fuel rises, the number of commuters using
public transportation will continue to decline.
C. The number of commuters using public transportation will rise, if the
price of fuel rises to a prohibitive level.
D. Most of the commuters using public transportation do not use fuel,
therefore fluctuations in the price of fuel is unlikely to affect the
number of commuters using public transportation.
E. The price of fuel is always low enough to make private
transportation cheaper than public transportation; therefore
fluctuations in the price of fuel is unlikely to affect the number of
commuters using public transportation.

Ans : C

72. Pharmaceutical firms are now engaged in the production of analogues of


endorphins, peptides, which are thought to transmit messages among
brain cells, which bring about pain relief. The pharmaceutical firms claim
that the analogues, when injected into the bloodstream, will lead to
effective and long-lasting pain relief by increasing the action of peptides
already present in the brain.

The claims put forth by the pharmaceutical firms would be considerably


weakened if it were true that

A. Compared to the other types of neurotransmitters present in the


brain, endorphins remain active for a longer time period.
B. Certain peptides have been traced in parts besides the brain, like
the alimentary canal and the skin.
C. It is easier and cheaper to produce analogues of peptides in the
laboratory than the peptides themselves.
D. Analogues of peptides, which are found naturally in the body are
often filtered from the blood before the blood circulates in the brain.
E. Endorphins interact chemically both with other naturally occurring
peptides and with the brain's other neurotransmitters.

Ans : D

73. Quite often it happens that a particular crisis or opportunity induces people
to find a practical use for things, which originally had no serious purpose.
To exemplify this principle, consider dolls and mannequins, programmed

25
to move and built for the delight of the wealthy in the eighteenth century;
these were the forerunners of the modern computer. Similarly, it is almost
certain that pets were the first domesticated animals. Domestication of
animals seems to have arisen as an amusement long before it had any
practical application.

Which among the following, provides another example supporting the


above mentioned principle, if true?

A. America was discovered as a by-product of the search for ginger,


cloves, pepper, and cinnamon.
B. Children often imitate adults in their games.
C. Till the commercial and scientific potential of the spyglass was
recognized and its power of magnification was improved it
remained a mere source of diversion.
D. Some culture horses were used only for pleasure and not for work,
even though in those cultures people had to labor hard in the
absence of laboring animals.
E. People who made moving dolls and mannequins in the eighteenth
century were also clockmakers.

Ans : C

74. Ever since the communications-entertainment firms have taken over


publishing concerns, the management's new methods have increased the
financial profits of commercial publishing. This is done at the price of
narrowing the range of available books to the public and by catering to the
vulgar tastes of the new buyers of books. There business has boomed ;
but in the process the losers are the majority of authors or aspiring writers,
and all discriminating readers.

In case the above statement is true, which among the following will hold
true?

A. Profitable business ventures are relatively new in the publishing


world.
B. The readership which commercial publishing caters to today is
different from the readership served in the past.
C. The profits resulting from the methods introduced by
communications-entertainment management will encourage literary
writers to work against all the odds.
D. The limited range of books available to the public is directed toward
a more discriminating audience.
E. The public is not aware of the trend in the publishing industry,
which tends to specialize in publishing blockbuster books.

26
Ans : B

75. It is popularly believed that teachers are more or less indifferent about the
microcomputer technology. This assumption is false, or at least dated. A
survey recently conducted indicated that 80 percent of the 7,000 surveyed
teachers revealed a high level of interest in microcomputers.

Among the following statements which would most damage the above
argument if proved to be true?

A. There was no attempt made in the survey to ascertain whether the


surveyed teachers had any previous exposure to microcomputers.
B. Teachers interested in microcomputer technology were more likely
to complete and return the questionnaires than others.
C. Irrespective of their subject area, their expertise and their teaching
experience questionnaires were received by the teachers.
D. After the survey results were tabulated there have been many
developments in the applications of microcomputer technology.
E. A company manufacturing and selling microcomputers conducted
the survey.

Ans : B

Questions 76 - 79

Six scientists A, B, C, D, E, and F are to present a paper each at a one-day


conference. Three of them will present their papers in the morning session before
the lunch break whereas the other three will be presented in the afternoon
session. The lectures have to be scheduled in such a way that they comply with
the following restrictions:

B's should present his paper immediately before C's presentation; their
presentations cannot be separated by the lunch break.
D must be either the first or the last scientist to present his paper.

76. In case C is to be the fifth scientist to present his paper, then B must be
A. first
B. second
C. third
D. fourth
E. sixth

Ans : D

77. B could be placed for any of the following places in the order of presenters
EXCEPT

27
A. first
B. second
C. third
D. fourth
E. fifth

Ans : C

78. In case F is to present his paper immediately after D presents his paper, C
could be scheduled for which of the following places in the order of
presenters?
A. First
B. Second
C. Third
D. Fourth
E. Fifth

Ans : E

79. In case F and E are the fifth and sixth presenters respectively then which
of the following must be true?
A. A is first in the order of presenters.
B. A is third in the order of presenters.
C. A is fourth in the order of presenters.
D. B is first in the order of presenters.
E. C is fourth in the order of presenters.

Ans : C

Questions 80 - 82

In a small inn, one or more of the chefs have to perform duty during
dinnereveryday. The chefs are Nicholas, Antonio, and Jeremy.

None of them can be assigned to dinner duty two or more days in a row.

80. In case Antonio and Jeremy share the dinner duty thrice over a five-day
period, which among the following would be true?
A. Nicholas is on dinner duty alone on the first of the five days.
B. Nicholas is on dinner duty alone on the second of the five days.
C. Nicholas is on dinner duty alone on the third of the five days.
D. Antonio and Jeremy share dinner duty on the second of the five
days.
E. Antonio and Jeremy share dinner duty on the fourth of the five
days.

28
Ans : C

81. In case Nicholas and Antonio share dinner duty on Monday of some week,
and if Antonio and Jeremy share dinner duty on Thursday of the same
week, which of the following would be true for that week?
A. Antonio is on dinner duty alone on Tuesday.
B. Jeremy is on dinner duty alone on Wednesday.
C. Nicholas and Jeremy share dinner duty on Wednesday.
D. Nicholas is on duty alone on Tuesday, and Antonio is on breakfast
duty alone on Wednesday.
E. Jeremy is on breakfast duty alone on Tuesday, and Nicholas is on
breakfast duty alone on Wednesday.

Ans : E

82. Which among the following could be true of some four-day period?
A. On three of the four days, pairs of chefs were on dinner duty.
B. On one of the four days, all three chefs were on dinner duty.
C. Antonio drew twice as many dinner assignments as did Jeremy.
D. Nicholas drew three times as many dinner assignments as did
Antonio.
E. Both Nicholas and Antonio drew three times as many dinner
assignments as did Jeremy.

Ans : C

83. Which among the following statements is the most probable taking into
consideration the source and the degree of seriousness with which they
are uttered?
A. Examiner: "The exam will be taken under the honor system: the
professors have the honor, and the students have the system."
B. Banker: "The low percentage of Jews in this bank has nothing to do
with discrimination. Jews don't seem to be interested in banking."
C. Book cover: "This book is sure to become the last word in its field."
D. Politician: "My opponent has an impeccable political and personal
record."
E. Artist: "Einstein's theory of relativity falsifies life."

Ans : D

84. Tom: It is likely that Greece will be humiliated in the Cyprus affair. In case
she fights Turkey, she will be defeated in battle; whereas if she doesn't
fight, letting Turkey win, she will be humiliated, as she would then seem
impotent.

Sarah: In that case Greece could always call the Soviet Union to her aid.

29
Sarah attacks Tom's argument by:

A. attacking Tom on a personal level.


B. attacking Tom's method of reasoning
C. siding with Greece
D. advocating passive resistance
E. citing an alternative which was overlooked by Tom.

Ans : E

85. In case Sue sits between Pete and Harry, then Harry sits between Sue
and Mike. Harry won't be there unless Sue sits next to Mike. Hence, Sue
will not sit between Pete and Harry.

Apart from the above mentioned statements, what additional premises are
assumed by the author of this argument?

Mike sits next to Sue if no one sits between them.


If Sue sits between Pete and Harry, then Sue sits between Harry and
Pete.

If Harry isn't there, then he doesn't sit next to Mike.

A. I and II only
B. I and III only
C. II and III only
D. I, II and III
E. None of the above

Ans : B

Questions 86 - 89

An increasing number of people prefer to retain their own individuality and their
own identity and consequently this has lead to a decline in the marriage rate.

86. Which among the following assumptions are used in the above premises?
I. When a person is married, he or she loses his or her own identity
and is no longer accountable to himself or herself.
II. Married persons do not find contentment as opposed to unmarried
people.
III. There has been a steady increase in the divorce rate.

A. I only
B. II only
C. III only

30
D. I and II only
E. I, II, and III

Ans : A

87. Among the following statements, which would weaken the above
argument?
A. The stability resulting from marriage offsets the negative aspects of
the dual responsibility of husband and wife.
B. Most people are not mature enough to be married.
C. Among most married couples the wife wants to have children.
D. There are a differing set of values honored by men and women.
E. It is advantageous to be single form a tax point of view.

Ans : A

88. Which among the following would strengthen the above argument?
A. Very few people prefer to bring up a family.
B. Emotionally divorce is not an easy procedure.
C. 700 couples from 1000 surveyed couples complained that they
were losing their identity.
D. Married people have to make a considerable effort to make the
marriage last.
E. The financial complications arising from a divorce are becoming
decreasingly complicated.

Ans : C

89. Emma: Financially the private university is ruined. In case it raises its
tuition fees, the number of paying students will come down and if they
refrain from raising the fees the university will go bankrupt.

Richard: The pay of the teachers and professors could be cut.


Richard responds to Emma’s argument by

A. drawing attention to an earlier overlooked alternative.


B. attacking her method of reasoning.
C. resigning himself to the demise of the private university.
D. attacking Emma on a personal level.
E. taking the side of the university as against that of the students.

Ans : A

Questions 90 - 92

31
Mrs. Green wishes to renovate her cottage. She hires the services of a plumber,
a carpenter, a painter, an electrician, and an interior decorator. The renovation is
to completed in a period of one working week i.e. Monday to Friday. Every
worker will be taking one complete day to do his job. Mrs. Green will allow just
one person to work per day.

1. The painter can do his work only after the plumber and the carpenter have
completed their jobs.
2. The interior decorator has to complete his job before that of the electrician.
3. The carpenter cannot work on Monday or Tuesday.

90. In case the painter works on Thursday, which among the following
alternatives is possible?
A. The electrician works on Tuesday.
B. The electrician works on Friday.
C. The interior decorator does his work after the painter.
D. The plumber and the painter work on consecutive days.
E. Mrs. Green cannot fit all of the workers into schedule.

Ans : B

91. In case the painter works on Friday, which among the following
statements must be untrue?
A. The carpenter may work on Wednesday.
B. The carpenter and the electrician may work on consecutive days.
C. In case the carpenter works on Thursday, the electrician has to
work on the previous day i.e. Wednesday.
D. The plumber may work before the electrician does.
E. The electrician may work on Tuesday.

Ans : C

92. Which arrangement among the following is possible?


A. The electrician will work on Tuesday and the interior decorator on
Friday.
B. The painter will work on Wednesday and the plumber on Thursday.
C. The carpenter will work on Tuesday and the painter on Friday.
D. The painter will work on Monday and the carpenter on Thursday.
E. The carpenter will work on Wednesday and the plumber on
Thursday.

Ans : E

93. According to the laws pertaining to the use of recyclable containers,


beverages are sold only in reusable containers. The people who object to
such laws point out that the collection of reusable bottles requires

32
gasoline, and washing them requires water; but, it takes less water to
wash these containers than it does to make throwaway bottles. The
expenditure saved due to the saved water exceeds the cost of the
gasoline used to transport empty bottles.

The above passage directly answers which of the below mentioned


objections to recyclable beverage containers?

A. It is likely that consumers will continue to discard even the


recyclable containers.
B. Consumers will find it an inconvenience to return bottles.
C. The extra expense involved in recycling would raise the prices of
beverages.
D. Recycling would be detrimental to the glass and plastic industries,
which produce containers.
E. Recycling of containers only partially answers the problem of
disposal of solid waste.

Ans : C

94. A recently conducted study reveals that the prospects for success in later
schooling for 3-5 old children were improved by a particular educational
plan. From this it follows that the introduction of similar education plans for
all 3-5 year old children will lead to an improvement in their chances for
success in later schooling.

Which among the following would weaken the above argument if true?

A. Parents of preschool children in the United States get attracted to


fads in the educational field, without having a clear idea of the type
of early educational plans which would benefit their children.
B. The cognitive abilities of children are constantly changing between
ages three and five
C. The people who conducted the research included quite a few
children who had earlier been exposed to another educational
enrichment program.
D. Many parents are of the view that formal training at an early age
takes up the time, which the children could better spend exploring
their worlds independently.
E. To establish such educational programs at a national level would
require extraordinary public expense.

Ans : C

Questions 95 - 97

33
Three girls Joan, Rita, and Kim and two boys Tim and Steve are the only dancers
in a dance program, which consists of six numbers in this order: One a duet; two
a duet; three a solo; four a duet; five a solo; and six a duet.

None of the dancers is in two consecutive numbers or in more than two numbers.

The first number in which Tim appears is the one that comes before the first
number in which Kim appears.
The second number in which Tim appears is one that comes after the second
number in which Kim appears.

95. Which among the following is a complete and accurate list of those
numbers that could be the last one in which Kim performs?
A. Three
B. Four
C. Five
D. Three, Four
E. Four, Five

Ans : E

96. Rita must perform only in duets if


A. Kim is in number two
B. Kim is in number five
C. Tim is in number one
D. Tim is in number two
E. Tim is in number six

Ans : D

97. In case Steve is in number five, number four must consist of


A. two women
B. two men
C. Tim and a woman
D. Rita and a man
E. Kim and a man

Ans : A

Refer to the test taking tips in the Logical Reasoning section. For the most part, they
apply to the quantitative section as well.

1. READ THE QUESTION CAREFULLY! Make sure you understand


exactly what you are being asked to do.
2. Use your calculator! It will save you time and effort, and help
you make accurate calculations.

34
3. Answers will often not work out to whole numbers or simple
fractional values, as may have been the case on other
standardized tests you have taken. This is another reason why
calculators are recommended for this assessment.
4. Use the scratch paper provided.
5. If you are forced to guess because you cannot solve a problem
or run out of time, do not be too quick to select the "none of the
above" option. This response is no more likely to be correct than
any of the other alternatives. Instead, try to rule out response
choices before you make your guess.
6. Review and practice working some problems in the topic areas
listed below, especially if you have not worked with the concepts
recently.

Q. 98 In general, the terms entrepreneur and SME’s (small and


medium enterprises) are used in conjunction with each other. But
entrepreneurship is not just about owning a small business. The very
basis of entrepreneurship, irrespective of the size of the organization,
lies in its ability to create new products/ services, and devising new
ways of implementing the existing or new products.

Which of the following best sums up the above passage?

(A) The major characteristic of entrepreneurs is their ability to


introduce new products/services into the market.

(B) Entrepreneurs are the main actors in economic growth.

(C) An owner of a large business may be an entrepreneur.

(D) Entrepreneurs do not own or operate small business.

35
Q.99: During the summer this year, advertising expenditures on soft
drinks increased by 25%, while the consumption of soft drinks
increased by 30%. Which of the following is irrelevant to explaining
the increase in consumption of soft drinks?

(A) This summer, soft drink companies offered more volume


discounts than competing substitutes.

(B) Soft drinks were available in more retail outlets this summer.

(C) The advertisements of soft drinks were more catchy and


effective this summer.

(D) The production of soft drink bottle openers doubled this year.

Instructions for Questions 100-101

A two-way road network exists between the following locations in a


city: A and B, A and C, C and E, E and G, E and H, G and D, and D
and F. There is also a one-way road between locations D and B; the
only possible way of travel is from D to B. None of these road routes
intersect each other except meeting at the nodal points in the
respective locations. There are no other routes to or from the above
locations in the city.

36
Q.100 Which of the following locations cannot be avoided while
traveling from F to H?

(A) B, C

(B) D, C

(C) D, E

(D) D, A

Q.101: What is the minimum number of locations one would have to


touch to reach E from F?

(A) 2

(B) 3

(C) 4

(D) 5

Q.102: The following sentences, when properly sequenced, form, a


coherent paragraph. Select the most logical order of the sentences.

37
(i) Transmission and Distribution losses are very high in Indian State
Electricity Boards. (ii) Electricity rates have to be raised. (iii) State
Electricity Boards in India are making commercial losses. (iv) High
technical losses lead to loss of revenue and subsequent rise in
electricity prices.

(A) (ii)-(i)-(iii)-(iv)

(B) (iii)-(i)-(ii)-(iv)

(C) (iii)-(i)-(iv)-(ii)

(D) (i)-(ii)-(iii)-(iv)

Q.103: Seven MBA students are to be assigned projects, as part of


their curriculum. Three students (A, B and C) are engineering
graduates, two (D and E) are science graduates, and two (F and G)
are commerce graduates. The course instructor has offered them
three projects coded here as 1, 2 and 3.

No student can take part in more than one project.

There must be atleast one engineering graduate in each project.

B cannot be in the same project as G.

C and F must work on the same project.

38
E must not work on project 3.

Since projects 1 and 2 are easier, the instructor has allowed only two
students to work on these projects.

Which of the following pairs CAN NOT work on the same project?

(A) C and G

(B) D and F

(C) D and E

(D) B and E

Topics covered in the Quantitative Part

You are expected to understand and use correctly a wide variety of basic mathematical
concepts covered in most introductory level college mathematics courses.

This is an assessment of your basic quantitative skills, not your knowledge of advanced
mathematics or economics. There is no calculus or matrix algebra, no econometrics or
economic theory on the assessment. Although you might be tested on your understanding
of linear regression, you will not be asked to develop any regression equations.

You should be prepared to solve problems involving such topics as:

 Decimals, fractions, and


 Probability
rounding procedures

39
 Percentage and percentage
 Basic statistics, including
change
-- Measures of central tendency
 Ratios and proportions
(mean, median, and mode)
 Inequalities -- Weighted average
-- Standard deviation, variance,
 Linear equations
and other measures of dispersion
 Non-linear functions (e.g.,
rate of change, growth rates)
 Graphs and charts

Question 104

From 1992 to 1998, 2,139 fatalities resulted from job-related aircraft accidents. Of these
fatalities, 853 were pilots. The other victims were "on the job" when they died and not
employed as pilots.

According to the Federal Aviation Administration, the leading cause of aviation accidents
is pilot error. During the period mentioned above, pilot error was cited as either a cause
or a contributing factor in 77% of all aviation accidents and in 83% of fatal aviation
accidents.

From these data it can be concluded that, during 1992 - 1998,

A) For aircraft accidents attributed to pilot error, the total number of non-
fatal aircraft accidents was 6% less than the total number of fatal aircraft
accidents

B) Slightly over 60% of fatalities resulting from job-related aircraft


accidents occurred to non-pilots

C) The total percentage of aircraft accidents for which pilot error cannot
be cited as a contributing factor (or cause) is approximately 17%

D) Less than 20% of all aircraft accidents involved no pilot fatalities

E) None of these

Which letter corresponds to the correct response? ______

Question 105

40
A) For aircraft accidents attributed to pilot error, the total number
of non-fatal aircraft accidents was 6% less than the total number of
fatal aircraft accidents

B) Slightly over 60% of fatalities resulting from job-related


aircraft accidents occurred to non-pilots

C) The total percentage of aircraft accidents for which pilot error


cannot be cited as a contributing factor (or cause) is approximately
17%

D) Less than 20% of all aircraft accidents involved no pilot fatalities

E) None of these

The correct response is B. According to the passage, there were 2,139 fatalities resulting
from job-related aircraft accidents during 1992-1998. Of these, 853, or approximately
39.88% (i.e., 853 ÷ 2,139), involved pilot fatalities. Therefore, as stated in response B,
slightly over 60% of fatalities (i.e., 100% - 39.88%) occurred to non-pilots.

Response A is incorrect. The figure 6% is obtained by subtracting 77% from 83% in the
question. But this subtraction is invalid because the two percentages are based on
different total numbers. The 77% figure refers to all accidents (fatal and non-fatal),
whereas the 83% refers to fatal accidents only.

Response C is also incorrect. The assertion in response C is that in approximately 17% of


aircraft accidents, pilot error was not cited as a contributing factor. Option C does not
limit the types of "aircraft accident". Therefore, it must refer to "all" aircraft accidents.
The 83% figure in the passage refers to fatal accidents only. Therefore, 17% can only
represent the percentage of fatal accidents not caused by pilot error. On the other hand,
the passage states that pilot error was a cause or factor in 77% of all accidents. From this
it follows that pilot error was not a factor in approximately 23% of all accidents.

The claim in response D that "less than 20% of all aircraft accidents involved no pilot
fatalities" again involves subtracting 83% from 100% to get 17%. This misinterprets the
83% figure as applying only to those accidents in which there were pilot fatalities. The
83% figure actually refers to the percentage of all fatal accidents where pilot error was a
contributing factor or cause.

Question 106

In July 1998, the following data were published for four categories of blue-collar workers
in private industry in Huntsville, Alabama.

41
Mean
Number of
Occupation hourly
workers
earnings

1. Precision production, craft,


7,235 $ 13.61
and repair occupations

2. Machine operators,
13,122 $ 14.08
assemblers and inspectors

3. Transportation an material
1,391 $ 10.87
moving occupations

4. Handlers, equipment
2,910 $ 9.05
clearners, helpers, and laborers

Based on these data, the mean hourly earnings, rounded to the nearest
cent, of a blue-collar worker who worked in private industry in Huntsville,
Alabama, in 1998 was

A) $ 11.90

B) $ 11.91

C) $ 13.16

D) $ 13.17

E) None of these

Which letter corresponds to the correct response? ______

Question 107

A) For aircraft accidents attributed to pilot error, the total number


of non-fatal aircraft accidents was 6% less than the total number of
fatal aircraft accidents

B) Slightly over 60% of fatalities resulting from job-related


aircraft accidents occurred to non-pilots

C) The total percentage of aircraft accidents for which pilot error


cannot be cited as a contributing factor (or cause) is approximately
17%

42
D) Less than 20% of all aircraft accidents involved no pilot fatalities

E) None of these

The correct response is B. According to the passage, there were 2,139 fatalities resulting
from job-related aircraft accidents during 1992-1998. Of these, 853, or approximately
39.88% (i.e., 853 ÷ 2,139), involved pilot fatalities. Therefore, as stated in response B,
slightly over 60% of fatalities (i.e., 100% - 39.88%) occurred to non-pilots.

Response A is incorrect. The figure 6% is obtained by subtracting 77% from 83% in the
question. But this subtraction is invalid because the two percentages are based on
different total numbers. The 77% figure refers to all accidents (fatal and non-fatal),
whereas the 83% refers to fatal accidents only.

Response C is also incorrect. The assertion in response C is that in approximately 17% of


aircraft accidents, pilot error was not cited as a contributing factor. Option C does not
limit the types of "aircraft accident". Therefore, it must refer to "all" aircraft accidents.
The 83% figure in the passage refers to fatal accidents only. Therefore, 17% can only
represent the percentage of fatal accidents not caused by pilot error. On the other hand,
the passage states that pilot error was a cause or factor in 77% of all accidents. From this
it follows that pilot error was not a factor in approximately 23% of all accidents.

The claim in response D that "less than 20% of all aircraft accidents involved no pilot
fatalities" again involves subtracting 83% from 100% to get 17%. This misinterprets the
83% figure as applying only to those accidents in which there were pilot fatalities. The
83% figure actually refers to the percentage of all fatal accidents where pilot error was a
contributing factor or cause.

Question 108

Game testers play games for a living. They play each and every part of a game in order to
identify possible errors in video game software, so the errors can be fixed before the
software is shipped to stores. In a recent year, testers found that among all the games that
they tested, 12% contained technical errors (for example, game characters who walked
through walls, and sound or artwork that disappeared unpredictably). Nine percent (9%)
contained errors in the game-play itself (e.g., portions that were too difficult, too easy, or
too confusing for members of the intended audience.) The testers found that these two
types of error were independent of one another.

From this information it follows that, if a game is selected at random from


all the games that were tested, the probability that it will be found to
contain both types of error

A) Cannot be estimated

43
B) Is between 0.01 and 0.02

C) Is equal to 0.12

D) Is between 0.19 and 0.20

E) None of these

Which letter corresponds to the correct response? ______

) Cannot be estimated

B) Is between 0.01 and 0.02

C) Is equal to 0.12

D) Is between 0.19 and 0.20

E) None of these

The correct response is B. The error rates of 12% (or 0.12) and 9% (or 0.09) apply to "all
the games that were tested" and so they can be used to solve the problem as posed. The
probability of the joint occurrence of two independent events (in this case, a technical
error and a game-play error) is equal to the product of the individual probabilities of
occurrence. In this case, the joint probability equals 0.12 times 0.09 or 0.0108. Therefore,
the probability value for the conjunction is between 0.01 and 0.02, which is within the
range given by response B.

Response A is incorrect in claiming that the probability cannot be estimated. The


estimation has just been demonstrated.

From the information given, the error in response C probably involves assuming that all
pieces of software with a game-play error (12%) also contain a technical error. This
would contradict the information given, which is that the two types of error occur
independently.

The task in question 3 is to determine the probability that both types of error will be
present, in a game selected at random from the defined set of all games that were tested.
Response D results from erroneously treating the task as one of calculating the
probability that either one event or the other, but not both, will occur. Accordingly, the
value given in response D is calculated by adding the individual probabilities of
occurrence for each of the two events and then subtracting the probability of their joint

44
occurrence, thus: 0.12 + 0.09 - 0.0108 = 0.1992. This, however, is not the result the
question asks for.

LECTURE 5-10

Quantitative Section : Quantitative Comparison

Directions:

In this section you will be given two quantities, one in column A and one in
column B. You are to determine a relationship between the two quantities and
mark.

A. If the quantity in column A is greater than the quantity in column B.


B. If the quantity in column B is greater than the quantity in column A.
C. If the quantities are equal.
D. If the comparison cannot be determined from the information that is given.

1. Quantity A: (-6)4
Quantity B: (-6)5
A. if the quantity A is greater;
B. if the quantity B is greater;
C. if the two quantities are equal;
D. if the relationship cannot be determined from the information given.

Ans : A

2. Quantity A: Time to travel 95 miles at 50 miles per hour


Quantity B: Time to travel 125 miles at 60 miles per hour
A. Quantity A is greater
B. Quantity A equals Quantity B
C. Quantity B is greater
D. Relationship Indeterminate

Ans : C

3. Quantity A: (9/13)2
Quantity B: (9/13)1/2
A. Quantity A equals Quantity B
B. Relationship Indeterminate
C. Quantity B is greater
D. Quantity A is greater

Ans : C

45
4. Quantity A: 4 / 100
Quantity B: 0.012 / 3
A. Quantity B is greater
B. Quantity A equals Quantity B
C. Quantity A is greater
D. Relationship Indeterminate

Ans : C

5. x = 2y + 3
y = -2

Quantity A: x
Quantity B: -1

A. if the quantity in Column A is greater;


B. if the quantity in Column B is greater;
C. if the two quantities are equal;
D. if the relationship cannot be determined from the information given

Ans : C

6. x + 2y > 8

Quantity A: 2x + 4y
Quantity B: 20

A. if the quantity in Column A is greater;


B. if the quantity in Column B is greater;
C. if the two quantities are equal;
D. if the relationship cannot be determined from the information given.

Ans : D

7. Quantity A: The number of months in 7 years


Quantity B: The number of days in 12 weeks
A. if the quantity in Column A is greater;
B. if the quantity in Column B is greater;
C. if the two quantities are equal;
D. if the relationship cannot be determined from the information given

Ans : C

46
8. Quantity A: 1-1/27
Quantity B: 8/9 + 1/81
A. if the quantity in is greater;
B. if the quantity in is greater;
C. if the two quantities are equal;
D. if the relationship cannot be determined from the information given.

Ans : A

9. r/>s/>0/>

Quantity A: rs/r
Quantity B: rs/s

A. if the quantity A is greater;


B. if the quantity B is greater;
C. if the two quantities are equal;
D. if the relationship cannot be determined from the information given.

Ans : B

10. Quantity A: 0.83


Quantity B: 0.81/3
A. Quantity B is greater
B. Relationship Indeterminate
C. Quantity A is greater
D. Quantity A equals Quantity B

Ans : A

11. t is a positive integer.


4/7 = t/s

Quantity A: s
Quantity B:7

A. if the quantity in Column A is greater;


B. if the quantity in Column B is greater;
C. if the two quantities are equal;
D. if the relationship cannot be determined from the information given

47
Ans : D

12. Quantity A: (0.82)2(0.82)3


Quantity B:(0.82)6
A. if the quantity in Column A is greater;
B. if the quantity in Column B is greater;
C. if the two quantities are equal;
D. if the relationship cannot be determined from the information given.

Ans : A

13. For all real numbers a, let a* = 1 - a.

Quantity A: ((-1)*)*
Quantity B: 2*

A. if the quantity in Column A is greater;


B. if the quantity in Column B is greater;
C. if the two quantities are equal;
D. if the relationship cannot be determined from the information given.

Ans : C

14. Quantity A: (x - 1)(x)(x + 1)


Quantity B:(x)(x)(x)
A. if the quantity in Column A is greater;
B. if the quantity in Column B is greater;
C. if the two quantities are equal;
D. if the relationship cannot be determined from the information given.

Ans : D

15. Quantity A: (3 x 4 x 17) / (121 x 100)


Quantity B: (4 x 5 x 19) / (1000 x 121)
A. Quantity A is greater
B. Quantity A equals Quantity B
C. Relationship Indeterminate
D. Quantity B is greater

48
16. Consider a triangle PQR.

Quantity A: length of PQ + length of QR


Quantity B: length of PR

A. Quantity A is greater
B. Quantity B is greater
C. Relationship Indeterminate
D. Quantity A equals Quantity B

Ans : A

17. Quantity A: (27 - 13) (296 + 534)


Quantity B: (27 + 13) (534 + 296)
A. Quantity B is greater
B. Quantity A equals Quantity B
C. Relationship Indeterminate
D. Quantity A is greater

Ans : D

18. Quantity A: A = 1.1


Quantity B: 12.11/2
A. Relationship Indeterminate
B. Quantity B is greater
C. Quantity A equals Quantity B
D. Quantity A is greater

Ans : B

19. 100 < y < 200 and 100 < z < 210

Quantity A: y
Quantity B: z

A. Quantity A is greater
B. Quantity A equals Quantity B
C. Quantity B is greater
D. Relationship Indeterminate

49
Ans : D

20. y2 + z2 = 34 and yz = 15

Quantity A: y2 + 2yz + z2
Quantity B: (y + z)2

A. Quantity B is greater
B. Relationship Indeterminate
C. Quantity A is greater
D. Quantity A equals Quantity B

Ans : D

21. Consider a rectangle. The length of its shorter side is 8, and the length of
its diagonal is 16.

Quantity A: 30o
Quantity B: measure of angle formed by diagonal and shorter side

A. Relationship Indeterminate
B. Quantity A equals Quantity B
C. Quantity A is greater
D. Quantity B is greater

Ans : D

22. Quantity A: (y + 5)2


Quantity B: (y - 5)2
A. Quantity B is greater
B. Relationship Indeterminate
C. Quantity A equals Quantity B
D. Quantity A is greater

Ans : B

23. Quantity A: (1/25)1/2 + (1/144)1/2


Quantity B: [(1/25) + (1/144)]1/2
A. Relationship Indeterminate
B. Quantity A is greater
C. Quantity B is greater
D. Quantity A equals Quantity B

50
Ans : A

24. y2 + z2 = 34 and yz = 15

Quantity A: y2 + 2yz + z2
Quantity B: (y + z)2

A. Quantity A is greater
B. Relationship Indeterminate
C. Quantity A equals Quantity B
D. Quantity B is greater

Ans : C

25. 100 < y < 200 and 100 < z < 210

Quantity A: y
Quantity B: z

A. Quantity A is greater
B. Quantity A equals Quantity B
C. Quantity B is greater
D. Relationship Indeterminate

Ans : D

26. Quantity A: (y + 5)2


Quantity B: (y - 5)2
A. Quantity A equals Quantity B
B. Quantity A is greater
C. Relationship Indeterminate
D. Quantity B is greater

Ans : C

27. Consider a rectangle. The length of its shorter side is 8, and the length of
its diagonal is 16.

Quantity A: 30o
Quantity B: measure of angle formed by diagonal and shorter side

A. Quantity A is greater

51
B. Quantity A equals Quantity B
C. Quantity B is greater
D. Relationship Indeterminate

Ans : C

28. The sum of three consecutive even numbers is 18.

Quantity A: Their average


Quantity B: 6

A. Relationship Indeterminate
B. Quantity A is greater
C. Quantity A equals Quantity B
D. Quantity B is greater

Ans : C

29. x - y > 10

Quantity A: y - x
Quantity B: 12

A. Quantity B is greater
B. Quantity A is greater
C. Quantity A equals Quantity B
D. Relationship Indeterminate

Ans : A

30. x = 0, y > 0

Quantity A: xy
Quantity B: yx

A. Quantity A equals Quantity B


B. Quantity A is greater
C. Quantity B is greater
D. Relationship Indeterminate

Ans : C

52
31.

Diagram is illustrative and is not drawn to scale.

Quantity A: Measure of angle 3 - Measure of angle 2


Quantity B: Measure of angle 5 - Measure of angle 6

A. Relationship Indeterminate
B. Quantity A equals Quantity B
C. Quantity B is greater
D. Quantity A is greater

Ans : B

32. Quantity A: 29
Quantity B: 92
A. Quantity B is greater
B. Quantity A is greater
C. Relationship Indeterminate
D. Quantity A equals Quantity B

Ans : B

33. 0 < -x < 10


11 < -y < 20

Quantity A: x
Quantity B: y

A. Relationship Indeterminate
B. Quantity A equals Quantity B
C. Quantity B is greater
D. Quantity A is greater

Ans : D

53
34.

Diagram is illustrative and is not drawn to scale.


Given angles 1 and 2 are equal,
length of side AB = x, length of side BC = y, length of side AC = z.

Quantity A: x + y
Quantity B: y + z

A. Quantity A is greater
B. Quantity A equals Quantity B
C. Quantity B is greater
D. Relationship Indeterminate

Ans : B

35.

Diagram is illustrative and is not drawn to scale.


In triangle ABC, AB = AC and measure of angle 1 = 100o.

Quantity A: Measure of angle 2 + Measure of angle 3


Quantity B: 90o

A. Quantity B is greater
B. Relationship Indeterminate
C. Quantity A is greater
D. Quantity A equals Quantity B

54
Ans : A

36. x and y are both positive and x/y > 5

Quantity A: 0.2x
Quantity B: y

A. Quantity A is greater
B. Quantity B is greater
C. Relationship Indeterminate
D. Quantity A equals Quantity B

Ans : A

37.

Diagram is illustrative and is not drawn to scale.


Given AB = AC and angle BAC = 60o

Quantity A: Length of side AB


Quantity B: Length of side BC

A. Quantity A equals Quantity B


B. Quantity B is greater
C. Relationship Indeterminate
D. Quantity A is greater

Ans : A

38. y2 = 36

Quantity A: y
Quantity B: 6

A. Relationship Indeterminate
B. Quantity A is greater
C. Quantity B is greater
D. Quantity A equals Quantity B

55
Ans : A

39.

Diagram is illustrative and is not drawn to scale.

Quantity A: Measure of angle 1 + Measure of angle 2 + Measure of angle


4
Quantity B: 180o

A. Relationship Indeterminate
B. Quantity A is greater
C. Quantity B is greater
D. Quantity A equals Quantity B

Ans : D

40.

Diagram is illustrative and is not drawn to scale.


In triangle ABC, angle A = 60o and AB = AC.

Quantity A: Measure of angle 1 + Measure of angle 2


Quantity B: 120o

A. Relationship Indeterminate
B. Quantity A is greater
C. Quantity A equals Quantity B
D. Quantity B is greater

Ans : C

56
41.

Diagram is illustrative and is not drawn to scale.

Quantity A: Measure of angle 2 + Measure of angle 3


Quantity B: 180o

A. Quantity B is greater
B. Quantity A is greater
C. Quantity A equals Quantity B
D. Relationship Indeterminate

Ans : C

42.

Diagram is illustrative and is not drawn to scale.


AB is the diameter of the circle.

Quantity A: Measure of angle 1


Quantity B: Measure of angle 2

A. Relationship Indeterminate
B. Quantity A is greater
C. Quantity B is greater
D. Quantity A equals Quantity B

Ans : D

57
43.

Diagram is illustrative and is not drawn to scale.

Quantity A: Measure of angle 1 + Measure of angle 3


Quantity B: Measure of angle 2 + Measure of angle 4

A. Relationship Indeterminate
B. Quantity A is greater
C. Quantity A equals Quantity B
D. Quantity B is greater

Ans : A

44.

Diagram is illustrative and is not drawn to scale.


In triangle ABC, AB = AC and measure of angle 1 = 100o.

Quantity A: Measure of angle 2 + Measure of angle 3


Quantity B: 90o

A. Quantity B is greater
B. Quantity A equals Quantity B
C. Relationship Indeterminate
D. Quantity A is greater

Ans : A

58
45.

Diagram is illustrative and is not drawn to scale.


Given angles 1 and 2 are equal,
length of side AB = x, length of side BC = y, length of side AC = z.

Quantity A: x + y
Quantity B: y + z

A. Quantity B is greater
B. Quantity A equals Quantity B
C. Quantity A is greater
D. Relationship Indeterminate

Ans : B

46. x and y are both positive and x/y > 5

Quantity A: 0.2x
Quantity B: y

A. Quantity B is greater
B. Relationship Indeterminate
C. Quantity A equals Quantity B
D. Quantity A is greater

Ans : D

47. yz < 0

Quantity A: (y - z)2
Quantity B: y2 + z2

A. Quantity A is greater
B. Quantity B is greater
C. Quantity A equals Quantity B
D. Relationship Indeterminate

59
Ans : A

48. For any positive integer n,


n! is the product of all positive integers less than or equal to n.

Quantity A: 20! / 17!


Quantity B: 80! / 78!

A. Quantity A is greater
B. Quantity B is greater
C. Quantity A equals Quantity B
D. Relationship Indeterminate

Ans : A

49. 2 < z < 4

Quantity A: π2z3
Quantity B: π3z2

A. Quantity A is greater
B. Quantity B is greater
C. Quantity A equals Quantity B
D. Relationship Indeterminate

Ans : D

50. Amy, Beth and Charlie divided a pizza amongst themselves.


Amy took 30% of the pizza and ate (3/4) of what she took.
Beth took 20% of the pizza.
Charlie ate (2/5) of what he took.

Quantity A: The amount Amy ate


Quantity B: The amount Charlie ate

A. Quantity A is greater
B. Quantity B is greater
C. Quantity A equals Quantity B
D. Relationship Indeterminate

Ans : A

51. p > 0 > q

Quantity A: p + q
Quantity B: pq

60
A. The quantity in Column A is greater.
B. The quantity in Column B is greater.
C. The quantities are equal.
D. The relationship cannot be determined from the information given.

Ans : D

52. Quantity A: The average (arithmetic mean) of x and y


Quantity B: The average (arithmetic mean) of x - 1 and y + 1
A. The quantity in Column A is greater.
B. The quantity in Column B is greater.
C. The quantities are equal.
D. The relationship cannot be determined from the information given.

Ans : C

53. The integer (x - 1) is a prime number between 40 and 50.

Quantity A: The sum of all different prime factors of x


Quantity B: 14

A. The quantity in Column A is greater.


B. The quantity in Column B is greater.
C. The quantities are equal.
D. The relationship cannot be determined from the information given.

Ans : B

54. A < C
B>D>0

Quantity A: A - B
Quantity B: C - D

A. The quantity in Column A is greater.


B. The quantity in Column B is greater.
C. The quantities are equal.
D. The relationship cannot be determined from the information given.

Ans : B

55. In a particular jellybean jar, the number of red jellybeans exceeds the
number of white ones by a ratio of 3:2. If two red jellybeans were
removed, the ratio of red to white jellybeans would be 1:1.

61
Quantity A: The number of white jellybeans in the jar
Quantity B: 4

A. The quantity in Column A is greater.


B. The quantity in Column B is greater.
C. The quantities are equal.
D. The relationship cannot be determined from the information given.

Ans : C

62
LECTURE 5-10

Quantitative Section : Quantitative Ability


Directions:In this section you will be given two quantities, one in column A and
one in column B. You are to determine a relationship between the two quantities
and mark.

A. If the quantity in column A is greater than the quantity in column B.


B. If the quantity in column B is greater than the quantity in column A.
C. If the quantities are equal.
D. If the comparison cannot be determined from the information that is given.

1. A rectangle is 14 cm long and 10 cm wide. If the length is reduced by x


cms and its width is increased also by x cms so as to make it a square
then its area changes by :
A. 4
B. 144
C. 12
D. 2
E. None of the above.

Ans : A

2. A motorcycle stunts man belonging to a fair, rides over the vertical walls of
a circular well at an average speed of 54 kph for 5 minutes. If the radius of
the well is 5 meters then the distance traveled is:
A. 2.5 kms
B. 3.5 kms
C. 4.5 kms
D. 5.5 kms
E. None of the above

Ans : C

3. If 1 cm on a map corresponds to an actual distance of 40 kms. And the


distance on the map between Bombay and Calcutta is 37.5 cms., the
actual distance between them is :
A. 375 kms
B. 3750 kms
C. 1500 kms
D. 1375 kms
E. None of the above

Ans : C

63
4. A box contains 90 mts each of 100 gms and 100 bolts each of 150 gms. If
the entire box weighs 35.5 kg., then the weight of the empty box is :
A. 10 kg
B. 10.5 kg
C. 11 kg
D. 11.5 kg
E. None of the above

Ans : D

5. If the radius of a circle is increased by 20% then the area is increased by :


A. 44%
B. 120%
C. 144%
D. 40%
E. None of the above

Ans : A

6. Tom, Dick and Harry went for lunch to a restaurant. Tom had $100 with
him, Dick had $60 and Harry had $409. They got a bill for $104 and
decided to give a tip of $16. They further decided to share the total
expenses in the ratio of the amounts of money each carried. The amount
of money which Tom paid more than what Harry paid is
A. 120
B. 200
C. 60
D. 24
E. 36

Ans : E

7. A plot of land is in the shape of a trapezium whose dimensions are given


in the figure below :

Hence the perimeter of the field is


A. 50 m
B. 64 m
C. 72 m
D. 84 m
E. None of the above

64
Ans : c

8. Four concentric ( having the same center ) circles with radii, x, 2x, 3x and
4x are drawn to form two rings A and B as shown in the figure.

Ratio of the area of inner ring A to the area of outer ring B is


A. 1 : 2
B. 1 : 4
C. 2 : 3
D. 3 : 7
E. None of the above

Ans : D

9. If 3/p = 6 and 3/q = 15 then p - q = ?


A. 1/3
B. 2/5
C. 3/10
D. 5/6
E. None of the above

Ans : C

10. A father is three times as old as his son. After fifteen years the father will
be twice as old as his son's age at that time. Hence the father's present
age is
A. 36
B. 42
C. 45
D. 48
E. None of the above

Ans : C

11. (1/4)3 + (3/4)3 + 3(1/4)(3/4)(1/4 + 3/4) =?


A. 1/64
B. 27/64
C. 49/64
D. 0

65
E. 1

Ans : E

12. If the area of two circles are in the ratio 169 : 196 then the ratio of their
radii is
A. 10 : 11
B. 11 : 12
C. 12 : 13
D. 13 : 14
E. None of the above

Ans : D

13. A semi-circle is surmounted on the side of a square. The ratio of the area
of the semi-circle to the area of the square is

A. 1:2
B. 2:p
C. p:8
D. 8:p
E. None of the above

Ans : C

14. Which of the following is the greatest ?


A. 40% of 30
B. 3/5 of 25
C. 6.5% of 200
D. Five more than the square of 3
E. 1/2-4

Ans : E

15. Two identical taps fill 2/5 of a tank in 20 minutes. When one of the taps
goes dry in how many minutes will the remaining one tap fill the rest of the
tank ?
A. 5 minutes
B. 10 minutes

66
C. 15 minutes
D. 20 minutes
E. None of the above

Ans : C

16. If the value of XYZ Company stock drops from $25 per share to $21 per
share, what is the percent of the decrease?
A. 4
B. 8
C. 12
D. 16
E. 20

Ans : D

17. If a building b feet high casts a shadow f feet long, then, at the same time
of day, a tree t feet high will cast a shadow how many feet long?
A. ft/b
B. fb/t
C. b/ft
D. tb/f
E. t/fb

Ans : A

18. If x, y, and z are consecutive negative integers, and if x > y > z, which of
the following must be a positive odd integer?
A. xyz
B. (x - y) (y - z)
C. x - yz
D. x(y + z)
E. x + y + z

Ans : B

19. At a certain ice cream parlor, customers can choose among five different
ice cream flavors and can choose either a sugar cone or a waffle cone.
Considering both ice cream flavor and cone type, how many distinct triple-
scoop cones with three different ice cream flavors are available?
A. 12
B. 16
C. 20
D. 24
E. 30

67
Ans : C

20. What is the greatest value of a positive integer n such that 3n is a factor of
1815?
A. 15
B. 18
C. 30
D. 33
E. 45

Ans : C

21. If .2t = 2.2 - .6s and .5s = .2t + 1.1, then s =


A. 1
B. 3
C. 10
D. 11
E. 30

Ans : B

22. Five years ago, Beth's age was three times that of Amy. Ten years ago,
Beth's age was one half that of Chelsea. If C repre- sents Chelsea's
current age, which of the following represents Amy's current age?
A. c/6 + 5
B. 2c
C. (c-10)/3
D. 3c-5
E. 5c/3 - 10

Ans : A

23. A portion of $7200 is invested at a 4% annual return, while the remainder


is invested at a 5% annual return. If the annual income from both portions
is the same, what is the total income from the two investments?
A. $160
B. $320
C. $400
D. $720
E. $1,600

Ans : B

24. An empty swimming pool can be filled to capacity through an inlet pipe in
3 hours, and it can be completely drained by a drainpipe in 6 hours. If both

68
pipes are fully open at the same time, in how many hours will the empty
pool be filled to capacity?
A. 4
B. 4.5
C. 5
D. 5.5
E. 6

Ans : E

25. If r = (3p + q)/2 and s = p - q, for which of the following values of p would
r2 = s2?
A. 1q/5
B. 10 - 3q/2
C. q - 1
D. 3q
E. 9q/2 - 9

Ans : A

26. At 10 a.m. two trains started traveling toward each other from stations 287
miles apart. They passed each other at 1:30 p.m. the same day. If the
average speed of the faster train exceeded the average speed of the
slower train by 6 miles per hour, which of the following represents the
speed of the faster train, in miles per hour?
A. 38
B. 40
C. 44
D. 48
E. 50

Ans : C

27. On the xy-coordinate plane, points A and B both lie on the circumference
of a circle whose center is O, and the length of AB equals the circle's
diameter. If the (x,y) coordinates of O are (2,1) and the (x,y) coordinates of
B are (4,6), what are the (x,y) coordinates of A?
A. (3, 3/2)
B. (1, 2/2)
C. (0, -4)
D. (2/2, 1)
E. (-1, -2/2)

Ans : C

69
28. If a rectangle's length and width are both doubled, by what percent is the
rectangle's area increased?
A. 50
B. 100
C. 200
D. 300
E. 400

Ans : D

29. A rectangular tank 10" by 8" by 4" is filled with water. If all of the water is
to be transferred to cube-shaped tanks, each one 3 inches on a side, how
many of these smaller tanks are needed?
A. 9
B. 12
C. 16
D. 21
E. 39

Ans : B

30. Point Q lies at the center of the square base (ABCD) of the pyramid
pictured above. The pyramid's height (PQ) measures exactly one half the
length of each edge of its base, and point E lies exactly halfway between
C and D along one edge of the base. What is the ratio of the surface area
of any of the pyramid's four triangular faces to the surface area of the
shaded triangle?
A. 3 :√2
B. √5:1
C. 4√3:3
D. 2√2:1
E. 8:√5

Ans : D

Quantitative Section : Data Interpretation


Questions31 -3 5 refers to the following table:

PROFILE OF CONGRESS IN YEAR X


(total membership: 535)

House of
Senate
Representatives
Party
292 Democratic 62

70
143 Republican 38
435 TOTAL 100
Sex
418 Male 100
17 Female 0
Age
27 Youngest 34
77 Oldest 80
Average
48 54
(arithmetic mean)
Religion
255 Protestant 69
107 Catholic 12
18 Jewish 5
4 Mormon 3
51 Other 11
House of
Senate
Representatives
Profession
215 Lawyer 63
Business Executive
81 15
or Banker
45 Educator 6
14 Farmer or Rancher 6
22 Career Government 0
24 Official 4
Journalist or
2 0
Communications
1 Executive 1
0 Physician 2
Veterinarian
6 0
Geologist
Worker or Skilled
25 Tradesperson 3
Other
Ethnic Group
17 Black American 1
2 Asian American 3
4 Hispanic American 0

31. In the Senate, if 25 male members were replaced by 25 female members,


the ratio of male members to female members would be
A. 4 to 1
B. 3 to 1
C. 3 to 2
D. 2 to 1

71
E. 1 to 1

Ans : B

32. Approximately what percent of the members of Congress are lawyers?


A. 63%
B. 58%
C. 56%
D. 52%
E. 49%

Ans : D

33. If 5 senators are Catholic Democrats, how many senators are neither
Catholic nor Democratic?
A. 79
B. 74
C. 69
D. 31
E. 21

Ans : D

34. If all lawyers and all women in the House of Representatives vote for the
passage of a bill, how many more votes will be needed for a majority?
A. 435
B. 220
C. 3
D. 0
E. It cannot be determined from the information given.

Ans : E

35. Which of the following can be inferred from the information given in the
chart?

I. More than 80 percent of the men in Congress are members of the


House of Representatives.
II. The percent of members who are categorized as farmers or
ranchers is greater for the House of Representatives than for the
Senate.
III. The median age in the Senate is 57.

A. I only
B. II only
C. III only

72
D. I and II
E. I and III

Ans : A

Section : Quantitative Ability

Q.36: A text book for children is meant to have 216 sq. cms. of actual
printed matter in each page. Also, the top and bottom margins are 3
cms. each and the left and right margins are 2 cms. each. The most
economical height and width of each page will be respectively

(A) 16 cms. and 12 cms.

(B) 20 cms. and 14 cms.

(C) 24 cms. and 16 cms.

(D) 18 cms. and 12 cms.

Q.37: The probability of a number being divisible by 3, not divisible


by 5 and divisible by either 4 or 6 is

(A) 1/6

(B) 2/15

73
(C) 1/30

(D) 5/6

Q.38. Let PQR be a right-angled triangle, right-angled at R, and let


RS be the perpendicular from R to PQ. Let PQ=a, QR=b, RP=c, RS=d
and PS=e. Which one of the following is not always true?

(A) ad=bc

(B) b2 + c2 = 2c2 + ½ a2

(C) 1/d2 =1/b2 + 1/c2

(D) d2 = e(a-e)

Q.39: Five students are participating in a contest. Three teams are


to be made in such a way that each student has to be a member of
one and only one team. However, team sizes need not be the same.
If the order of the teams or the order of the students within the
teams does not matter, the number of ways in which three teams can
be formed is

(A) 35

(B) 7

(C) 20

74
(D) 25

Q.40: Any complex number x + iy can be put in the form r( cosθ +


isinθ ) where r is called the modulus and θ is the argument of the
complex number. The complex number z having the least positive
argument and satisfying | z – 5i | ≤ 3 is

(A) 9/5 + i (15/5)

(B) 2/5 + i (3/5)

(C) 12/5 + i (16/5)

(D) none of the above

Q. 41: The angle of elevation of a kite from a point 100 meters above
a lake is 30O and the angle of depression of its reflection in the lake is
60O. The height of the kite above the lake is

(A) 100 √3/3 meters

(B) 200 √3 meters

(C) 100 meters

(D) 200 meters

75
Section : Data Interpretation

Questions 42 –45 are based on the above graph, which depicts the
performance parameters of LMN Ltd, for ten years.

Q.42: In which of the following years did LMN Ltd. suffer a loss?

(A) 1991-92, 1992-93, 1993-94, 1994-95

(B) 1990-91, 1991-92, 1992-93, 1993-94

(C) 1993-94, 1994-95, 1995-96

76
(D) 1991-92, 1992-93, 1994-95

Q.43: The highest percentage of growth in total revenue was


recorded between the financial years

(A) 1999-00 and 2000-01

(B) 1997-98 and 1998-99

(C) 1992-93 and 1993-94

(D) 1994-95 and 1995-96

Q.44: If the percentage growth rate between the financial years


1999-00 and 2000-01 was maintained, the total revenue for the year
2001-02 would approximately be

(A) Rs. 192.30 millions.

(B) Rs. 171.87 millions.

(C) Rs. 174.74 millions.

(D) Rs. 164.41 millions.

77
Q.45: In which year was the Net Profit at its highest as a percentage
of total revenue?

(A) 1989-90

(B) 2000-01

(C) 1999-00

(D) 1990-91

DIRECTIONS

Directions for Problem Solving Questions


(Questions 46,47,50,52,53,55)

Solve each problem and indicate the best of the answer choices
given.

All numbers used are real numbers.

Directions for Data Sufficiency Questions


(Questions 48,49,51,54)
Each Data Sufficiency problem consists of a question and two
statements, labeled (1) and (2), in which certain data are given.
You have to decide whether the data given in the statements are
sufficient for answering the question. Using the data given in the
statements plus your knowledge of mathematics and everyday
facts, you must indicate whether:

78
o statement (1) ALONE is sufficient, but statement (2) alone
is not sufficient to answer the question asked;
o statement (2) ALONE is sufficient, but statement (1) alone
is not sufficient to answer the question asked;
o BOTH statements (1) and (2) TOGETHER are sufficient to
answer the question asked; but NEITHER statement ALONE
is sufficient;
o EACH statement ALONE is sufficient to answer the question
asked;
o statements (1) and (2) TOGETHER are NOT sufficient to
answer the question asked, and additional data specific to
the problem are needed.

Note: In data sufficiency problems that ask you for the value of
a quantity the data given in the statements are sufficient only
when it is possible to determine exactly one numerical value for
the quantity.

All numbers used are real numbers.

Question 46

If the average cost of producing one widget decreases from


$12.50 to $10.75, what is the percent of the decrease?

(A) 10
(B) 12.5
(C) 14
(D) 15
(E) 16

Question 47

(A) 1........... (B) ...... (C)

(D) ...... (E) 25

Question 48

79
Is p + q > pq ?

(1) p > 0 > q

(2) |q| = p
(A)
Statement (1) ALONE is sufficient to answer the question, but statement (2)
alone is NOT sufficient.
(B)
Statement (2) ALONE is sufficient to answer the question, but statement (1)
alone is NOT sufficient.
(C)
BOTH statements (1) and (2) TOGETHER are sufficient to answer the
question, but NEITHER statement ALONE is sufficient.
(D)
Each statement ALONE is sufficient to answer the question.
(E)
Statements (1) and (2) TOGETHER are NOT sufficient to answer the question.
Question 49

What is the average (arithmetic mean) of x and y?


(1) The average (arithmetic mean) of (x + 1) and (y - 1) is equal to the average
(arithmetic mean) of x and y.
(2) The average (arithmetic mean) of x, y, and -1 is 1.

(A)
Statement (1) ALONE is sufficient to answer the question, but statement (2)
alone is NOT sufficient.
(B)
Statement (2) ALONE is sufficient to answer the question, but statement (1)
alone is NOT sufficient.
(C)
BOTH statements (1) and (2) TOGETHER are sufficient to answer the
question, but NEITHER statement ALONE is sufficient.
(D)
Each statement ALONE is sufficient to answer the question.
(E)
Statements (1) and (2) TOGETHER are NOT sufficient to answer the question.
Question 50

If a building b feet high casts a shadow f feet long, then, at the


same time of day, a tree t feet high will cast a shadow how
many feet long?

(A) ...... (B) ...... (C)

D) ..... (E)

Question 51

80
What is the value of the integer K?
(1) K is a prime number between 40 and 50.
(2) The integer (K + 1) is divisible by exactly two different prime numbers.

(A)
Statement (1) ALONE is sufficient to answer the question, but statement (2)
alone is NOT sufficient.
(B)
Statement (2) ALONE is sufficient to answer the question, but statement (1)
alone is NOT sufficient.
(C)
BOTH statements (1) and (2) TOGETHER are sufficient to answer the
question, but NEITHER statement ALONE is sufficient.
(D)
Each statement ALONE is sufficient to answer the question.
(E)
Statements (1) and (2) TOGETHER are NOT sufficient to answer the question.
Question 52

If x, y, and z are consecutive non-zero integers, and if x < y < z,


which of the following must be a positive odd integer?

(A) x-y-z
(B) xy + z
(C) x+y-z
(D) xz - y
(E) x+y+z

Question 53

Two canoe riders must be selected from each of two groups of


campers. One group consists of three men and one woman, and
the other group consists of two women and one man. What is
the probability that two men and two women will be selected?

(A) ...... (B) ...... (C)

(D) ....... (E)

Question 54

At a garage sale, Jeff sold 80% of his books, which include both
hardbacks and paperbacks. He sold an equal number of each of
the two types of books, selling all paperbacks for $1 each and all
hardbacks for $3 each. How many books did Jeff own before the
sale?

81
(1) Jeff's total revenue from the sale of his books was $32.
(2) Exactly two hardbacks and two paperbacks remained unsold after the sale.

(A)
Statement (1) ALONE is sufficient to answer the question, but statement (2)
alone is NOT sufficient.
(B)
Statement (2) ALONE is sufficient to answer the question, but statement (1)
alone is NOT sufficient.
C)
BOTH statements (1) and (2) TOGETHER are sufficient to answer the
question, but NEITHER statement ALONE is sufficient.
D)
Each statement ALONE is sufficient to answer the question.
(E)
Statements (1) and (2) TOGETHER are NOT sufficient to answer the question.
Question 55

What is the greatest value of a positive integer n such that 3n is


a factor of 1815 ?

(A) 15
(B) 18
(C) 30
(D) 33
(E) 45

Answer Key
To return to a question, click on the answer.

46. (C)

47. (B)

48. (C)
49. (B)
50. (A)
51. (E)
52. (D)
53. (E)
54. (D)
55. (C)

82
LECTURE 10-15

DIRECTIONS

Directions for Problem Solving Questions


(Questions 1, 2, 5, 7, 8, and 10)

Solve each problem and indicate the best of the answer choices
given.

All numbers used are real numbers.

Directions for Data Sufficiency Questions


(Questions 3, 4, 6, and 9)
Each Data Sufficiency problem consists of a question and two
statements, labeled (1) and (2), in which certain data are given.
You have to decide whether the data given in the statements are
sufficient for answering the question. Using the data given in the
statements plus your knowledge of mathematics and everyday
facts, you must indicate whether:

o statement (1) ALONE is sufficient, but statement (2) alone


is not sufficient to answer the question asked;
o statement (2) ALONE is sufficient, but statement (1) alone
is not sufficient to answer the question asked;
o BOTH statements (1) and (2) TOGETHER are sufficient to
answer the question asked; but NEITHER statement ALONE
is sufficient;
o EACH statement ALONE is sufficient to answer the question

83
asked;
o statements (1) and (2) TOGETHER are NOT sufficient to
answer the question asked, and additional data specific to
the problem are needed.

Note: In data sufficiency problems that ask you for the value of a
quantity the data given in the statements are sufficient only when
it is possible to determine exactly one numerical value for the
quantity.

All numbers used are real numbers.

Question 1

If a + b - c = d, and if a - b + c = e, then a =

(A) (d + e)
(B) d-e
(C) 2d + e
(D) d+e
(E) 2(d + e)

Question 2

Five years ago, Beth's age was three times that of Amy. Ten
years ago, Beth's age was one half that of Chelsea. If C
represents Chelsea's current age, which of the following
represents Amy's current age?

(A) ...... (B) 2C ...... (C)

(D) 3C – 5 ...... (E)

Question 3

If the city of Franklin lies along a certain route between the cities
of Maple Valley and Viewcrest, what is the distance along the
route from Franklin to Maple Valley?

84
(1) Traveling from Viewcrest to Maple Valley at an average rate of 45 m.p.h.
takes 20 minutes longer than at an average rate of 50 m.p.h.
(2) Traveling along the route at a constant 25 miles per gallon of gas, a car will
use twice as much gas between Maple Valley and Franklin as it will use
between Franklin and Viewcrest.

(A)
Statement (1) ALONE is sufficient to answer the question, but statement (2)
alone is NOT sufficient.
(B)
Statement (2) ALONE is sufficient to answer the question, but statement (1)
alone is NOT sufficient.
(C)
BOTH statements (1) and (2) TOGETHER are sufficient to answer the
question, but NEITHER statement ALONE is sufficient.
(D)
Each statement ALONE is sufficient to answer the question.
E)
Statements (1) and (2) TOGETHER are NOT sufficient to answer the question.
Question 4

If A, B, C, and D are all positive numbers, is the value of A – B


greater than the value of C – D?
(1) A + D = B + C
(2) A and B are each greater in value than either C or D.

(A)
Statement (1) ALONE is sufficient to answer the question, but statement (2)
alone is NOT sufficient.
(B)
Statement (2) ALONE is sufficient to answer the question, but statement (1)
alone is NOT sufficient.
(C)
BOTH statements (1) and (2) TOGETHER are sufficient to answer the
question, but NEITHER statement ALONE is sufficient.
(D)
Each statement ALONE is sufficient to answer the question.
(E)
Statements (1) and (2) TOGETHER are NOT sufficient to answer the question.
Question 5

A portion of $6600 is invested at a 5% annual return, while the


remainder is invested at a 3% annual return. If the annual
income from the portion earning a 5% return is twice that of the
other portion, what is the total income from the two investments
after one year?

(A) $180
(B) $270
(C) $300
(D) $320
(E) $360

85
(B)

Question 6

Of all sophomore students attending a certain college, 20% are


enrolled in an English course while 10% are enrolled in a
psychology course. How many sophomore students attend the
college?
(1) 5% of the sophomore students at the college are enrolled in both an English
course and a psychology course.
(2) A total of 210 sophomore students at the college are enrolled in neither an
English course nor a psychology course.

(A)
Statement (1) ALONE is sufficient to answer the question, but statement (2)
alone is NOT sufficient.
(B)
Statement (2) ALONE is sufficient to answer the question, but statement (1)
alone is NOT sufficient.
(C)
BOTH statements (1) and (2) TOGETHER are sufficient to answer the
question, but NEITHER statement ALONE is sufficient.
(D)
Each statement ALONE is sufficient to answer the question.
(E)
Statements (1) and (2) TOGETHER are NOT sufficient to answer the question.
Question 7

If one typist can type a certain manuscript in 8 minutes, while


another typist can type the same manuscript in 12 minutes, how
long would it take both typists, working simultaneously, to type
the manuscript?

(A) 3 minutes, 36 seconds


(B) 4 minutes, 48 seconds
(C) 5 minutes, 15 seconds
(D) 6 minutes
(E) 10 minutes

Question 8

If r = and s = p – q, for which of the following values of p


would r = s2 ?
2

(A) ...... (B) 10 – ...... (C) q – 1

86
(D) 3q ........ (E) –9

Question 9

What is the value of x?


(1)
x+y=3

(2)
y+x=2

(A)
Statement (1) ALONE is sufficient to answer the question, but statement (2)
alone is NOT sufficient.
(B)
Statement (2) ALONE is sufficient to answer the question, but statement (1)
alone is NOT sufficient.
(C)
BOTH statements (1) and (2) TOGETHER are sufficient to answer the
question, but NEITHER statement ALONE is sufficient.
(D)
Each statement ALONE is sufficient to answer the question.
(E)
Statements (1) and (2) TOGETHER are NOT sufficient to answer the question.
Question 10

If eight pounds of macadamia nuts, priced at $6.00 per pound,


are combined with twelve pounds of brazil nuts, priced at $5.00
per pound, what is the per-pound price of the resulting mixture?

(A) $5.25
(B) $5.40
(C) $5.50
(D) $5.75
(E) $5.80

Answer Key
To return to a question, click on the answer.

1. (A)
2. (A)
3. (C)
4. (A)

87
5. (B)
6. (C)
7. (B)
8. (A)
9. (E)
10. (B)

OFFICIAL TEST DIRECTIONS

Directions for Problem Solving Questions


(Questions 11,12,15,17,18, and 20)

Solve each problem and indicate the best of the answer choices
given.

All numbers used are real numbers.

Directions for Data Sufficiency Questions


(Questions 13, 14, 16, and1 9)
Each Data Sufficiency problem consists of a question and two
statements, labeled (1) and (2), in which certain data are given.
You have to decide whether the data given in the statements are
sufficient for answering the question. Using the data given in the
statements plus your knowledge of mathematics and everyday
facts, you must indicate whether:

o statement (1) ALONE is sufficient, but statement (2) alone


is not sufficient to answer the question asked;
o statement (2) ALONE is sufficient, but statement (1) alone
is not sufficient to answer the question asked;
o BOTH statements (1) and (2) TOGETHER are sufficient to
answer the question asked; but NEITHER statement ALONE
is sufficient;
o EACH statement ALONE is sufficient to answer the question
asked;
o statements (1) and (2) TOGETHER are NOT sufficient to
answer the question asked, and additional data specific to
the problem are needed.

Note: In data sufficiency problems that ask you for the value of a
quantity the data given in the statements are sufficient only when
it is possible to determine exactly one numerical value for the

88
quantity.

All numbers used are real numbers.

Question 11

....

If AB and AC, inscribed in a square as shown above, are each 10


centimeters in length, what is the area of the square (in
centimeters)?

(A)
(B) 74
(C) 80
(D)
(E) 100

Question 12

If a rectangle's length and width are both doubled, by what


percent is the rectangle's area increased?

(A) 50
(B) 100
(C) 200
(D) 300
(E) 400

Question 13

89
....

In the figure above, is ?

(1) x + y = 180
(2) w = z

(A)
Statement (1) ALONE is sufficient to answer the question, but statement (2)
alone is NOT sufficient.
(B)
Statement (2) ALONE is sufficient to answer the question, but statement (1)
alone is NOT sufficient.
(C)
BOTH statements (1) and (2) TOGETHER are sufficient to answer the
question, but NEITHER statement ALONE is sufficient.
(D)
Each statement ALONE is sufficient to answer the question.
(E)
Statements (1) and (2) TOGETHER are NOT sufficient to answer the question.
Question 14

....

What is the value of x in the figure above?

(1) y + z = 140
(2) w + x = 80

(A)
Statement (1) ALONE is sufficient to answer the question, but statement (2)
alone is NOT sufficient.
(B)
Statement (2) ALONE is sufficient to answer the question, but statement (1)
alone is NOT sufficient.
(C)
BOTH statements (1) and (2) TOGETHER are sufficient to answer the
question, but NEITHER statement ALONE is sufficient.

90
(D)
Each statement ALONE is sufficient to answer the question.
(E)
Statements (1) and (2) TOGETHER are NOT sufficient to answer the question.
Question 15

In a certain parallelogram, the degree measure of one angle


exceeds that of another angle by 30. What is the degree
measure of the smaller angle?

(A) 30
(B) 55
(C) 60
(D) 70
(E) 75

Question 16

80 linear feet of fencing material is used to construct a


rectangular enclosure. What is the area of the enclosure?
(1) The length of one particular side of the enclosure is 10 feet.
(2) It is possible to divide the enclosure into three square enclosures of equal size.

(A)
Statement (1) ALONE is sufficient to answer the question, but statement (2)
alone is NOT sufficient.
(B)
Statement (2) ALONE is sufficient to answer the question, but statement (1)
alone is NOT sufficient.
(C)
BOTH statements (1) and (2) TOGETHER are sufficient to answer the
question, but NEITHER statement ALONE is sufficient.
(D)
Each statement ALONE is sufficient to answer the question.
(E)
Statements (1) and (2) TOGETHER are NOT sufficient to answer the question.
Question 17

On the xy-coordinate plane, all of the following xy-coordinate


points lie on the circumference of a circle whose radius is 10 and
whose center lies at the (x,y) point (-1,0) EXCEPT:

(A) (–6, –7)


(B) (–2, )
(C) (–1, –10)
(D) (7, 6)
(E) (1, – )

Question 18

91
....

Note: Figure not drawn to scale...

Point Q lies at the center of the square base (ABCD) of the


pyramid pictured above. The pyramid's height (PQ) measures
exactly one half the length of each edge of its base, and point E
lies exactly halfway between C and D along one edge of the
base. What is the ratio of the surface area of any of the
pyramid's four triangular faces to the surface area of the shaded
triangle?

(A) 3 : ......... (B) :1....... (C) :3

(D) 2 :1........... (E) 8 :

Question 19

....

In the figure above, does x = 90?

(1) The length of AC is less than the length of BC.


(2) The length of AB is one-fourth the circumference of the circle.

(A)
Statement (1) ALONE is sufficient to answer the question, but statement (2)
alone is NOT sufficient.
(B)
Statement (2) ALONE is sufficient to answer the question, but statement (1)
alone is NOT sufficient.
(C)
BOTH statements (1) and (2) TOGETHER are sufficient to answer the

92
question, but NEITHER statement ALONE is sufficient.
(D)
Each statement ALONE is sufficient to answer the question.
(E)
Statements (1) and (2) TOGETHER are NOT sufficient to answer the question.
Question 20

How many complete tanks of water, each with a capacity of 3


cubic meters, are needed to fill an empty cylindrical tank whose
height is 3 meters and whose base has a radius of 2 meters?

(A) 12
(B) 13
(C) 14
(D) 15
(E) 16

Answer Key
To return to a question, click on the answer.

1. (C)
2. (D)
3. (C)
4. (D)
5. (E)
6. (D)
7. (A)
8. (D)
9. (B)
10. (B)

DIRECTIONS

Directions for Problem Solving Questions

Solve each problem and indicate the best of the answer choices
given.

All numbers used are real numbers.

Questions21 and 22 refer to the following table:

93
Question 21

How much more does it cost to send a parcel weighing 16.5


pounds by express delivery than to send the same parcel by
ground delivery?

(A) $3.45
(B) $9.00
(C) $11.55
(D) $15.00
(E) $16.60

Question 22

By approximately what percent does the average air delivery


cost per pound for a 21-pound parcel exceed the average ground
delivery cost per pound for a 28-pound parcel?

(A) 33
(B) 46
(C) 67
(D) 80
(E) 130

Questions 23 and 24 refer to the following graph:

94
Question 23

Marketing expenses trailed production expenses by about the


same percentage during which of the following two years?

(A) 1990 and 1992


(B) 1991 and 1992
(C) 1991 and 1993
(D) 1992 and 1993
(E) 1990 and 1994

Question 24

Consider aggregate legal and production expenses for each year


shown. With respect to the two years during which this
aggregate amount was most nearly the same, average annual
marketing expenses totaled approximately

(A) $5.5 million


(B) $6.1 million
(C) $6.5 million
(D) $7.2 million
(E) $8.0 million

Answer Key
To return to a question, click on the answer.

21. (D)
22. (C)
23. (A)
24. (B)

95
96

Vous aimerez peut-être aussi